¿Muestra este experimento que la Ley de Kirchhoff se cumple cuando hay un campo magnético cambiante involucrado en un circuito?

En este video , el ingeniero eléctrico y youtuber Mehdi Sadaghdar (ElectroBOOM) discrepa con otro video del profesor Walter Lewin.

Básicamente, el profesor Lewin muestra en un experimento que si tenemos dos resistencias diferentes conectadas en un circuito cerrado, y si generamos un campo magnético variable usando una bobina, el voltaje en los extremos de las dos resistencias será diferente, contrariamente a lo esperado. de la Ley de Voltaje de Kirchhoff (KVL).

esquemático

simular este circuito : esquema creado con CircuitLab

Según el experimento, el voltímetro izquierdo VM1 muestra un voltaje diferente del segundo voltímetro VM2. Lewin luego concluye que KVL no se sostiene cuando hay un campo magnético cambiante. La razón matemática que da es que el campo magnético no es conservativo y KVL puede derivarse de las ecuaciones de Maxwell solo cuando el campo es conservativo. Luego dice que este experimento es una prueba de sus afirmaciones.

Mehdi, por otro lado, señala dos cosas: primero, que la forma en que se realizó el sondeo es incorrecta. El campo magnético cambiante tiene un efecto sobre los cables de la sonda, y esa es una de las razones por las que los voltímetros cambian de valor según la posición.

En segundo lugar, dice que debido a que hay un bucle, el bucle se comporta como un inductor y, junto con la bobina, forma un inductor mutuo:

esquemático

simular este circuito

Entiendo la derivación de Lewin del KVL, así que entiendo que hay un problema con el campo magnético no conservativo, pero al mismo tiempo creo que Mehdi tiene razón: ese bucle es un inductor, y la forma en que Lewin está probando el circuito parece incorrecta para a mí. Entonces, ¿dónde está el error aquí?

  • ¿KVL se mantiene en el circuito de arriba?
  • ¿Se está haciendo bien el sondeo?
  • ¿El circuito tiene un inductor mutuo que no debe ignorarse?
Usted sabe que la Ley de Faraday funciona y que Lewin tiene razón en el sentido de que la corriente en el bucle debe ser la misma en todo el bucle. Uno puede debatir sobre una configuración experimental, por supuesto. Pero la ley permanece.
No confunda el diagrama con el circuito real. Las leyes de Kirchoff describen componentes ideales en un modelo de componentes concentrados . Un circuito real tiene resistencia en lugares donde no hay un componente "resistor", inductancias (incluidas las inductancias mutuas ) en lugares donde no hay un componente "inductor", etc. Si los llamados componentes "parásitos" son importantes, entonces se acabó. a usted para representarlos explícitamente en su modelo (es decir, en su diagrama de circuito).
@jonk: tienes razón. La pregunta aquí (desde mi perspectiva) es si la configuración experimental es lo suficientemente precisa o no. O viceversa: si el modelo es lo suficientemente preciso para describir el experimento.
@SolomonSlow: de nuevo, estoy de acuerdo. El hecho aquí es que tenemos una persona (Lewin) que combina el diagrama con el circuito real (al menos, así es como lo entiendo). De ahí mi pregunta: ¿tiene razón al decir que el diagrama describe correctamente el circuito? ¿Importan los componentes parasitarios en este caso? Recuerde: no solo jugó con números, mostró un experimento y dijo que prueba sus afirmaciones.
Todo esto no tiene nada que ver con la verdad de KVL, es un error de ingeniería de medición para principiantes. Un voltímetro muestra la diferencia de voltaje entre dos puntos en su circuito interno . No muestra la diferencia de voltaje entre los extremos de las sondas que están conectadas. ¡Fin del problema! Por supuesto, en muchas situaciones, si usa el medidor de manera sensata, la diferencia entre el voltaje que se muestra y el voltaje entre las puntas de la sonda es demasiado pequeña para preocuparse. Pero en esta situación, ese no es el caso.
@HeyHey Depende de lo que quieras decir con "lo suficientemente preciso". Como en mi comentario anterior, el problema es simplemente que el circuito no mide lo que Lewin dice que mide. Eso es cierto, ya sea que los medidores sean o no "precisos" en el sentido de mostrar correctamente lo que realmente miden.
Todo esto no es diferente en principio de tratar de medir la corriente con un medidor cuya resistencia interna es del mismo orden de magnitud que la resistencia del circuito bajo prueba. Comprender por qué eso no funciona es un ejercicio en la mayoría de los cursos de "Circuitos de CC 101". En esa situación, nadie intentaría afirmar que el medidor es "inexacto" o que las leyes de Kirchhoff o de Ohm son "incorrectas". La explicación es simplemente que no estás midiendo lo que querías medir.

Respuestas (4)

Los modelos de componentes agrupados a los que se aplica KVL son solo eso: modelos. Como todos los modelos, solo son precisos en la medida en que representan las características relevantes del sistema que reflejan. El modelo de bucle simple de dos resistencias no representa la susceptibilidad de la ruta conductora que constituye el circuito a la FEM inducida, por lo tanto, este modelo simple no reflejará el comportamiento del circuito real en el mundo real donde la FEM inducida es algo que sucede.

El modelo simple se puede hacer más preciso al incluir inductores entre las resistencias y un inductor adicional que representa el solenoide que proporciona el campo magnético cambiante. Al considerar el acoplamiento de estos inductores, es posible incorporar la FEM inducida en el modelo y así lograr resultados que reflejen mejor la realidad. Un modelo razonablemente completo de la situación en la demostración de Lewin sería algo como lo siguiente ( fuente ), que también es lo que muestra Mehdi Sadaghdar. Tenga en cuenta que los resultados de la simulación de este modelo de elementos concentrados se parecen mucho a los de la demostración de Lewin.ingrese la descripción de la imagen aquí

Esta idea de refinar un modelo de circuito teórico agregando elementos agrupados para representar términos parásitos (es decir, características inherentes de un sistema que no son intencionales pero son relevantes para el comportamiento del sistema) no es exclusiva de situaciones donde hay un campo magnético cambiante, y es de hecho una práctica común y útil en ingeniería eléctrica. Por ejemplo, el comportamiento de un interruptor MOSFET se puede modelar con mayor precisión al incluir elementos para representar C GS y C GD .

En este caso, los inductores representan un fenómeno eléctrico que se rige por la relación física entre los elementos del circuito del mundo real. Como tal, si el circuito se reorganiza físicamente, los inductores del modelo deben ajustarse para reflejar las características eléctricas de esta nueva relación física. Este también es un aspecto bien entendido de la ingeniería eléctrica, donde, por ejemplo, la proximidad física de dos pistas en una PCB debe entenderse como algo que afecta la forma en que interactúan las señales en esas dos pistas.

En cierto punto, cuando las tasas de cambio en el estado del circuito se vuelven rápidas con respecto al tamaño físico de los componentes del circuito (¡incluidos los cables/pistas de PCB!), el elemento agrupado se vuelve difícil de manejar en el mejor de los casos e impreciso en el peor. en ese punto entran en juego cosas como los modelos de línea de transmisión, pero el modelo agrupado sigue siendo bastante útil en sistemas dinámicos que funcionan bien en el rango de MHz.

Entonces, en general, la afirmación de Lewin de que KVL no funciona para la situación que demuestra es básicamente correcta, pero solo porque el modelo de circuito utilizado no representa elementos que son cruciales para comprender su comportamiento en el mundo real.

Como nota al margen, puede parecer que Lewin no entiende lo que está sucediendo en este circuito, sin embargo, claramente lo entiende cuando examina el lenguaje específico que usa en la conferencia y en otros materiales. De este suplemento:

Suponga que coloca las puntas de prueba de un voltímetro en los terminales de un inductor (con una resistencia muy pequeña) en un circuito. ¿Qué vas a medir? Lo que medirá en el medidor del voltímetro es una "caída de voltaje" de Ldi/dt. ¡Pero eso no se debe a que haya un campo eléctrico en el inductor! Esto se debe a que colocar el voltímetro en el circuito dará como resultado un flujo magnético que cambia en el tiempo a través del circuito del voltímetro, que consiste en el inductor, los cables del voltímetro y la gran resistencia interna en el voltímetro.

Esto deja en claro que Lewin considera al voltímetro y sus cables como parte del circuito, y como ha dicho, el camino tomado a través del campo cambiante afecta la integral y por lo tanto el voltaje indicado por el medidor. Este es precisamente el efecto que Mehdi Sadaghdar describe en su video, solo observado desde una perspectiva física (Faraday et al) en lugar de una perspectiva EE (inductancias parásitas). No estoy seguro de por qué Lewin no ha optado por reconocer esta equivalencia, aparte de que considera que esta última es una "respuesta correcta por las razones equivocadas".

Editar para agregar:

En este video , Lewin expresa más claramente su objeción a formular el problema de una manera que refleje KVL. Para este circuito:

esquemático

simular este circuito : esquema creado con CircuitLab

Lewin muestra que, comenzando en la esquina inferior izquierda y moviéndose en el sentido de las manecillas del reloj, la integral de lazo cerrado de mi . d yo es como sigue (tenga en cuenta que no se muestra ningún término para el inductor porque se supone que es ideal, es decir, superconductor):

mi . d yo = V 0 + yo R + q C

Debido a estas dos identidades:

mi . d yo = d Φ B d t

d Φ B d t = L d yo d t

Podemos describir el circuito usando esta ecuación:

V 0 + yo R + q C = L d yo d t

Si quisiéramos obtener algo que se asemeje a KVL, simplemente podemos mover el término que describe V L al otro lado de la ecuación:

V 0 + yo R + q C + L d yo d t = 0

De esta última forma, Lewin dice que mover el término de inductancia hacia la izquierda "no hace que la ecuación sea incorrecta, ¡pero la física apesta!" porque ahora ninguno de los lados de la ecuación representa completamente mi . d yo .

Mhhhh, así lo explica en un suplemento. Me pregunto porque no lo explica en todos sus videos, y al contrario insiste en que el voltaje que mide es el correcto

¿KVL se mantiene en el circuito de arriba?

Sí. Ley de voltaje de Kirchhoff formulada por Kirchhoff en su artículo de 1845 "Ueber den Durchgang eines electrischen stromes etc." estados (mi traducción)

  1. cuando los cables 1 , 2 , . . . norte formar una figura cerrada,
    yo 1 R 1 + yo 2 R 2 + . . . yo norte R norte
    = la suma de todas las fuerzas electromotrices que están en el camino:

Cuando definimos la FEM inducida a lo largo de cualquier camino C por un campo magnético variable B como

mi i norte d tu C mi d = C mi r o t d

donde mi r o t es la componente libre de divergencia (o rotacional) de la mi campo, que es una solución a la ecuación

× mi r o t = B t

entonces, KVL se puede aplicar sin problemas al circuito anterior.

[Sin embargo, debe tenerse en cuenta que, al determinar la lectura de un voltímetro, también debemos incluir la EMF inducida en los cables].

Cabe señalar que KVL a menudo se enseña, no en la forma en que Kirchhoff lo expresó, sino en alguna variación de la siguiente forma:

La diferencia de potencial entre dos puntos cualesquiera de un circuito es independiente de la trayectoria. [Pseudo-KVL]

La declaración anterior, que llamo Pseudo-KVL, cuando es cierta, implica KVL. Además, KVL implica Pseudo-KVL cuando no hay inducción de un campo eléctrico por un campo magnético variable en el tiempo. Sin embargo, KVL y Pseudo-KVL tienen diferencias importantes.

KVL es una ley de bucle. Con esto quiero decir que describe una relación matemática que se encuentra al atravesar un bucle. Pseudo-KVL es una "ley" "potencial". Con esto quiero decir que asume esencialmente la existencia de diferencias de potencial. Todo eso está bien y elegante siempre que existan diferencias potenciales. Se vuelve problemático si la caída de voltaje a través de un componente depende de la ruta .

[Si se define "diferencia de potencial" como

a b mi d

entonces uno se encuentra con problemas cuando mi no es un campo conservativo. La integral toma diferentes valores dependiendo del camino entre a y b ]

¿Se está haciendo bien el sondeo?

No puede haber un sondeo "malo" en esta situación, porque no existe una forma "correcta" única de sondear. Diferentes arreglos de los cables de la sonda darán resultados diferentes, y esos resultados deben interpretarse . El dispositivo de medición, junto con sus dos cables de sonda, forman parte de un bucle. Cualquier ruta en el circuito bajo prueba entre los puntos de prueba forma otra parte de un bucle. Juntos, el bucle parcial que contiene el dispositivo de medición y el bucle parcial que contiene la parte del circuito bajo prueba, forman un bucle completo. De acuerdo con KVL, la suma de todos los EMF en ese bucle es igual a la suma de todas las caídas de voltaje IR en ese bucle. Por lo tanto, el voltaje a través del dispositivo de medición es igual a la suma de todos los campos electromagnéticos menos la suma de todas las caídas de voltaje IR que no seanel propio dispositivo de medición.

[Tenga en cuenta que dado que un dispositivo de medición (en una configuración fija) informará su medición de una manera determinada, no debería importar qué ruta dentro del circuito bajo prueba elegimos entre los puntos de prueba para resolver la ecuación KVL.]

¿El circuito tiene un inductor mutuo que no debe ignorarse?

Uno debe tener en cuenta los campos electromagnéticos inducidos en los cables. No importa cómo dibujemos el esquema. En algunos casos, puede ser más conveniente incluir cuatro inductores, uno para cada segmento de cable entre una resistencia y un punto de prueba. O bien, podemos omitir los símbolos del inductor, siempre que quede claro en el contexto que hay un campo magnético variable en el tiempo y que debemos tener en cuenta los campos electromagnéticos inducidos en los segmentos de cable al realizar el análisis del circuito. Contexto, contexto, contexto. Solo recuerde que, de una forma u otra, se debe tener en cuenta la EMF inducida en los cables.

Déjame copiar lo que comenté en el video. Por supuesto que "Lewin" tiene razón; Es física muy básica.

En la segunda parte de su video, básicamente explicó por qué no se puede definir un voltaje y por qué Lewin tiene razón. El punto exacto de un voltaje es que no debería importar cómo lo pruebe, debería ser el mismo de cualquier manera. La definición de voltaje es potencial eléctrico, es decir, la diferencia de voltaje entre dos puntos debería darte la energía total necesaria para mover una carga de un punto al otro, sin importar el camino. Si el camino importa, entonces todo se desmorona; El campo es no conservativo. Por supuesto, puede modelar estos efectos de diferentes maneras, como introducir un transformador, pero esos son solo eso, modelos, con limitaciones y siempre debe saber con qué limitaciones su modelo funciona como se espera.

ACTUALIZACIÓN: Veo que algunos de ustedes están un poco confundidos/perdidos. Déjame intentar y ayudar. Esta es la definición de voltaje en palabras (copiada de wikipedia):

Voltaje, diferencia de potencial eléctrico, presión eléctrica o tensión eléctrica es la diferencia de potencial eléctrico entre dos puntos. La diferencia de potencial eléctrico entre dos puntos (es decir, voltaje) se define como el trabajo necesario por unidad de carga contra un campo eléctrico estático para mover una carga de prueba entre los dos puntos.

Entonces, mueves una unidad de carga de un punto a otro y no importa el camino que hayas elegido para hacerlo , la energía total que necesitas para mover la carga de un punto al otro es la diferencia de voltaje entre los dos puntos. .

Ahora, lo que realmente dice la ley de Kirchhoff es que si tomas una carga en un viaje, pero en el y llevas la carga de regreso al punto de partida, el trabajo total que has realizado sobre la carga será 0. Desde aquí puedes vea fácilmente que no se mantendrá si la curvatura del campo eléctrico no es 0 en todas partes; porque puedes entrar en un bucle en el que E siempre apunta en la dirección opuesta de la marcha y cuando vuelvas al punto de partida, habrás hecho mucho trabajo contra el campo, aunque hayas llegado de nuevo al punto de partida primitivo.

Por ejemplo, en el bucle anterior (R1-R2) puede seguir moviéndose de un lado a otro y el trabajo realizado por usted aumentará monótonamente.

Si rotE no es idénticamente cero, no se puede definir un campo potencial, no se puede definir el voltaje (no existe), por lo que ni siquiera se puede hablar de voltaje en ningún contexto. Y la presencia de un campo magnético cambiante hace que E tenga un rizo, según la ecuación de Maxwell-Faraday.

El punto exacto de un voltaje es que no debería importar cómo lo pruebe, debería ser el mismo de cualquier manera. Cierto, pero si ignora la influencia de cómo lo prueba, puede engañarse a sí mismo. En mi opinión, eso lleva a Lewin a una conclusión equivocada. Por supuesto, la "física básica" siempre es cierta al igual que KVL, pero si ignora ciertos efectos, saca conclusiones equivocadas.
Ni siquiera es que KVL no sea cierto, sino que no tiene sentido. No existe el concepto de voltaje si rotE no es 0. Es como tratar de hablar sobre el siguiente número después de n con respecto a los números reales; Esto no tiene sentido. Por supuesto que una medición puede tener errores si no se hace con instrumentos ideales o con cuidado. Sin embargo, medir el voltaje a través de la definición arrojaría el mismo resultado; los "problemas" que el video identifica con el sondeo son los problemas exactos por los que no se puede definir el voltaje (potencial eléctrico).
Esta sería una buena respuesta... si nos limitamos a los modelos. Pero Lewin mostró un experimento del mundo real para probar sus afirmaciones, no se limitó a las leyes de la física. Cuando pasas de la teoría al mundo real, la forma en que mides las cantidades importa. La masa, como el voltaje, no cambia con la forma en que la mides, pero no puedes medirla poniéndote una escala en la cabeza.
Pero el experimento que hizo/hace es perfecto; Como dije, no tiene problema si la presuposición es que existe voltaje como concepto, los instrumentos están mostrando exactamente lo que está tratando de medir: la integral de E a lo largo del camino de las líneas de prueba, que es la definición de la diferencia de potencial eléctrico. Luego concluye que la integración a lo largo de diferentes caminos pero entre los mismos puntos da resultados diferentes, por lo que no se puede definir el voltaje entre los dos puntos. No hay problema con este argumento, que tiene más de un siglo, podría agregar.
Pero el experimento que hizo/hace es perfecto. ¿Cómo puede concluir que las partes no retorcidas del cable ignoran el campo magnético? No, no lo hacen, por lo que deben tenerse en cuenta. Lewin no lo hace y ese es su concepto erróneo.
@Bimpelrekkie No "ignoran" B, el par trenzado tampoco lo ignora. Los afectos de la B cambiante son el punto exacto. No sé qué más decir en este momento. Parece que estoy hablando con una pared: el voltaje (si está definido) entre 2 puntos es la integral de E a lo largo de un camino (CUALQUIER camino) de esos 2 puntos. El experimento mostrado por Lewin hace exactamente eso; Integra entre los mismos 2 puntos en caminos diferentes. Obtiene resultados diferentes, concluye que son diferentes, por lo tanto concluye que el voltaje entre los 2 puntos no existe.
@Cerike: el voltaje (si está definido) entre 2 puntos es la integral de E a lo largo de un camino (CUALQUIER camino) de esos 2 puntos. [...] Obtiene resultados diferentes, concluye que son diferentes, por lo tanto concluye que el voltaje entre los 2 puntos no existe. Esta es en realidad una excelente respuesta, mucho mejor que el wot :)
"La Ley de Kirchhoff realmente dice que si tomas una carga en un viaje, pero en el y llevas la carga de vuelta al punto de partida, el trabajo total que has hecho sobre la carga será 0". No, Kirchhoff no lo hizo. dilo. Otros dijeron eso. Lo que dijo Kirchhoff se encuentra en mi respuesta.
@Cerike "Ni siquiera es que KVL no sea cierto, sino que no tiene sentido. No existe el concepto de voltaje si rotE no es 0". KVL como se enseña podría no tener sentido cuando E es rotacional, pero KVL como se enseña no es lo que escribió Kirchhoff . Mira mi respuesta.

¿KVL se mantiene en el circuito de arriba?

Eso depende de cómo enmarque KVL. Creo que es seguro decir que uno debe asumir que está definido para un campo magnético uniforme, o posiblemente que está definido en un mundo mágico donde las líneas en una página son en realidad conductores perfectos sin resistencia y sin acoplamiento magnético o electrostático a otras líneas en la misma u otras páginas.

Tenga en cuenta que no estoy cagando KVL, pero se limita a exploraciones teóricas de circuitos ideales. Siempre debe tener en cuenta cómo sus circuitos reales van a diferir de la representación ideal en su esquema.

¿Se está haciendo bien el sondeo?

Esa es una pregunta de opinión. "Correcto" depende de lo que esté tratando de averiguar o de lo que esté tratando de probar.

¿El circuito tiene un inductor mutuo que no debe ignorarse?

Como se dibuja en el diagrama superior, sí. Pero tan pronto como colocas esa bobina allí, estás agregando elementos al esquema que no encajan con las suposiciones clásicas de los esquemas. De hecho, está rompiendo implícitamente una suposición clásica de los esquemas: que puede mover componentes arbitrariamente siempre que las líneas permanezcan conectadas. Al dibujar esa bobina allí, está tomando un diagrama esquemático perfectamente bueno y convirtiéndolo en un dibujo mecánico lamentablemente subespecificado.

Creo que el segundo dibujo le permitirá calcular con precisión los voltajes y las corrientes en las resistencias, pero para representar con precisión el efecto en los voltímetros necesitaría dos inductancias mutuas más, entre la bobina y el bucle de la resistencia y los cables de los medidores.

"Correcto" depende de lo que esté tratando de averiguar => bueno, creo que está claro en los videos lo que están tratando de averiguar: el voltaje en R1 y el voltaje en R2. No creo que haya una opinión en este caso.